Presentation is loading. Please wait.

Presentation is loading. Please wait.

AP Review Projects 1 st Period. Explanation: Negative Feedback is a primary mechanism of homeostasis, the condition of the body in which internal elements,

Similar presentations


Presentation on theme: "AP Review Projects 1 st Period. Explanation: Negative Feedback is a primary mechanism of homeostasis, the condition of the body in which internal elements,"— Presentation transcript:

1 AP Review Projects 1 st Period

2 Explanation: Negative Feedback is a primary mechanism of homeostasis, the condition of the body in which internal elements, including water, energy, temperature, and many different molecules, are regulated and maintained at a certain constant level, often labeled a set point. Homeostasis can also be defined as the steady-state physiological condition of the body because the body works to keep its internal environment constant using negative feedback. In a negative feedback loop, a positive or negative change in the physiological variable which is being monitored triggers a response by the body which counteracts the initial change. For example, in the figure to the right, a change in the blood calcium level above or below the set point causes either the thyroid gland to release calcitonin to decrease the amount of blood calcium or the parathyroid gland to release PTH to increase the amount of blood calcium. The change from the set point of blood calcium is detected by the thyroid or parathyroid gland, eliciting the glands to respond in their distinct way. The response of the effector (thyroid gland) reduces the initial stimulus, and the response eventually ceases once the original set point is reached. While the physiological variable being monitored may drift slightly above and below the set point due to a time lag between reception and response, negative feedback mechanisms prevent small changes from being too large from the norm. In another example, a cell’s breakdown of sugar to generate ATP may result in excess ATP, inhibiting an enzyme near the beginning of the cellular respiration pathway. Other examples of animal homeostatic processes regulated by negative feedback loops include body temperature, the endocrine and nervous system pathways, and blood glucose levels. An example of plant homeostatsis can be shown in a plant’s regulation of its water level. An excess or lack of water will cause the stomata to respond by closing to prevent the uptake of water or opening to allow more water in, respectively. Learning Objective 2.16: The student is able to connect how organisms use negative feedback to maintain their internal environments. Science Practice 7.2: The student can connect concepts in and across domain(s) to generalize or extrapolate in and/or across enduring understandings and/or big ideas. The general flow of negative feedback loops. http://study.com/academy/lesson/what-is-negative-feedback-in-biology-definition-examples.html http://hdwalls.xyz/images/homeostasis-pregnancy-positive-feedbackjpg An example to show how negative feedback loops are involved in homeostasis. MC Question: When the temperature of the body exceeds the set point of 37 ⁰ C, what responses would you expect the body to initiate to cool itself down? I.Sweating II.Circulating blood towards the skin III.Speeding up metabolism a) I only b) I and II only c) I and III only d) II and III only e) I, II, and III FRQ: Throughout a human’s day, sugar is constantly converted into ATP through cellular respiration to generate chemical energy for the body, which has a multitude of uses. When the body goes into sleep mode, however, the production of ATP changes. a)What happens to the production of ATP when a human falls asleep? Why? b)Explain the process(es) that allow for this change to occur. c)Why is it evolutionarily advantageous for this change to happen?

3 FRQ: Throughout a human’s day, sugar is constantly converted into ATP through cellular respiration to generate chemical energy for the body, which has a multitude of uses. When the body goes into sleep mode, however, the production of ATP changes. a)What happens to the production of ATP when a human falls asleep? Why? b)Explain the process(es) that allow for this change to occur. c)Why is it evolutionarily advantageous for this change to happen? Answers: a)The production of ATP decreases to a minimum because the body does not need energy while it sleeps. b)In a negative feedback loop, excess ATP that is made as the body falls asleep will not be used for any chemical processes and have nowhere to go. This will trigger a response to decrease the amount of ATP being made by having a protein bind to an enzyme near the beginning of the cellular respiration pathway. The inhibition of the enzyme prevents cellular respiration, and ATP will not be made, allowing the level of ATP excess to remain at the homeostatic minimum. c)If the body did not decrease the production of ATP while it sleeps, then ATP would be continuously made, resulting in a huge excess of ATP. This would waste resources, requiring the body to take in a much larger amount of food to power cellular respiration by supplying sugar. It is a selective advantage to need less food for survival. MC Question: When the temperature of the body exceeds the set point of 37 ⁰ C, what responses would you expect the body to initiate to cool itself down? I.Sweating II.Circulating blood towards the skin III.Speeding up metabolism a) I only b) I and II only c) I and III only d) II and III only e) I, II, and III Explanation: There are generally three responses that the body can initiate to cool itself down. It can sweat, releasing heat along with bodily fluids. It can circulate blood toward the skin so that heat is more likely to be given off through the selectively permeable barrier of the skin. The final response is that it can slow down metabolism so that less heat is made as a byproduct of metabolism. Speeding up metabolism, along with shivering and circulating blood toward the core of the body, would heat the body rather than cool it down. Answer Key:

4 LO 1.30: The student is able to evaluate scientific hypotheses about the origin of life on Earth. SP 6.5: The student can evaluate alternative scientific explanations. Explanation: There are many theories about the origin of life on planet Earth, but no definitive answer. The theory of panspermia is the idea that life on Earth evolved from microbes on rocks and other debris that landed on the planet. Another possibility is that life began near deep-sea vents, which are rich in hot water, minerals, and inorganic sulfur and iron compounds, essential factors in ATP production still. For decades, the most widely accepted theory has been that of chemosynthesis. A. I. Oparin and J. B. S. Haldane independently hypothesized that the environment of early Earth supported and facilitated the development of several organic compounds from extremely simple molecules in a “primordial soup,” along with energy for the reactions being provided from lightning and ultraviolet radiation from the sun. The 1953 experiment done by Stanley Miller and Harold Urey at the University of Chicago used an apparatus (see figure) to reflect the supposed early Earth atmosphere of the Oparin-Haldane hypothesis and several amino acids were able to be created in addition to some other organic compounds. From the monomers created in the Miller-Urey experiment and other similar ones, polymers could be formed by dripping them onto hot sand, as demonstrated by Sydney Fox. From then on, primitive protobionts, or abiotically produced molecules surrounded by a membrane, could by made. These protobionts were then subjected to natural selection. Those best suited for their environment would survive and be able to reproduce asexually. With the proper conditions, the protobionts would have been able to take in linked amino acids in their membranes, and made them selectively permeable to only a few organic molecules. Ribozymes are RNA catalysts and could have produced the first genetic material, RNA, which was selected for or against based on its structure and compatability with the environment. Eventually these factors would lead to autotrophic bacteria, which would, in turn, lead to more complex organisms in the future Multiple Choice: Which of the following is not true of protobionts, some of the earliest precursors to prokaryotic cells? A) They exhibited a very simple metabolism. B) They reproduced imprecisely with many mistakes and mutations from generation to generation. C) They had primitive nuclei. D) They had primitive membranes. Free Response: The environment of early Earth contained no living organisms, yet, somehow, life on Earth erupted, began to flourish, and eventually evolved into complex multicellular organisms such as humans. a)Describe the process of abiogenesis (i.e., when life naturally arises from non-living matter) on early Earth until the formation of the first single-celled organisms. b)Describe and explain the first experiment to model this process of the evolution of life from non-living matter.

5 Multiple Choice Answer: Which of the following is not true of protobionts, some of the earliest precursors to prokaryotic cells? A) They exhibited a very simple metabolism. B) They reproduced imprecisely with many mistakes and mutations from generation to generation. C) They had very simple nuclei. D) They had primitive membranes. Explanation) C is the correct answer because protobionts would not have had any sort of membrane-bound organelles, most notably a nucleus. It would not make sense from an evolutionary standpoint for precursors to prokaryotic cells, which have no membrane-bound organelles, to have a nucleus. Free Response: The environment of early Earth contained no living organisms, yet, somehow, life on Earth erupted, began to flourish, and eventually evolved into complex multicellular organisms such as humans. a)Describe the process of abiogenesis (i.e., when life naturally arises from non-living matter) on early Earth until the formation of the first single- celled organisms. b)Describe and explain the first experiment to model this process of the formation of the first organic compounds that led to the eruption of life on Earth. Answer/explanation for part a) Began with “primordial soup” of non-living matter; formed amino acids and other compounds which eventually formed polymers when they came into contact with each other and hot sand/clay; protobionts were formed from these polymers as they grew in complexity and either reproduced or did not survive based on their fitness in the environment (through natural selection; these protobionts led to the first single-celled organisms. Answer/explanation for part b) The Miller-Urey experiment; set up an apparatus that modeled the primitive atmosphere of early Earth; boiling water created water vapor that combined with gases presumed to have been in the primitive atmosphere and then further combined with a spark of electricity to model lightning which created the amino acids, purines, pyrimidines, and other compounds that are the building blocks of life.

6 LO 3.20: The student is able to explain how the regulation of gene expression is essential for the processes and structures that support efficient cell function SP 6.2: The student can construct explanations of phenomena based on evidence produced through scientific practices Explanation: DNA is stored wrapped around histones. This “package” is known as a nucleosome and they bind to neighboring nucleosomes, causing them to bunch up. This is good for the storage of DNA in the nucleus, but this makes it difficult for transcription factors to access genes. To make the genes more accessible the histones must be acetylated, allowing our cells that all contain the exact same genetic makeup to specialize and perform their different functions necessary for our body to work properly. Different cells also produce different regulatory factors, determining which genes get released. Genetic sequences must have promoting sequences at the beginning of it for the RNA Polymerase to bind to in order to transcribe that genetic sequence. Enhancers can be located further up on the DNA that determine the transcription of genes. Activators are then produced that attach to these enhancers to then bind to transcription factors and create a transcription initiation complex on the promoter of the gene sequence. The specific activators made in a cell determines which genes are expressed. Once the mRNA is transcribed, snRNPs and other proteins form together to form spliceosomes which cut out segments of the mRNA, introns, that are excess nucleotides that are not needed. The spliceosomes created by the cell then piece back together the remaining regions of the mRNA, the exons, to synthesize the desired amino acid. Another way for the expression of genes to be regulated is by repressor proteins. These proteins are made to bind to the promoter of a gene in order to prevent the transcription of that gene. A repressible operon is a gene that the product of which will bind to the protein when there is an excess produced. This will cause the protein to change shape and allow it to bind to the promoting site of the gene. An inducible gene that is constantly repressed by the active repressor protein except for when an outside stimuli causes the protein to change shape, preventing it from binding to the promoter site of the gene. Jordan Harm 1 st Period MC: What would happen if a protein bonded to the promoter site of the gene sequence, blocking the DNA polymerase from binding to it, that produces microtubules? A)This would not affect the cell B)The cell would not be able to perform mitosis C)This would speed up the process of mitosis D)This would increase the transmission of hormones from the cell Figure 19.5 Eukaryotic gene transcript FRQ: scientists isolated human digestive tract cells and placed them in a petri dish. To find out the significance of snRNPs in the function of a cell they then injected the cells with a drug that has been proven to prevent the snRNPs from activating. A)How would this affect the cells? B)What part of gene expression would not be affected by this drug? C) What would the scientists need to include in their experimental design for this experiment to provide accurate results

7 MC Question: What would happen if a protein bonded to the promoter site of the gene sequence, blocking the DNA polymerase from binding to it, that produces microtubules? A)This would not affect the cell B)The cell would not be able to perform mitosis C)This would speed up the process of mitosis D)This would increase the transmission of hormones from the cell B) The cell would not be able to perform mitosis Explanation: if a protein bonded to the promoter sight of the gene sequence, the microtubules would not be produced. This would prevent mitosis from occurring because microtubules are crucial in the separation of the two nuclei and cell membranes Answer Key A)This would prevent the cells from performing their proper functions. The proper polypeptide chains would not be created, because without the snRNPs the introns, excess nucleotides, will not be taken out of the final mRNA. These introns will then code for amino acids that are not necessary, preventing those polypeptide chains from functioning properly B)The DNA Polymerase would still bind to the promoting site of the gene sequence and transcribe it to create the pre- mRNA. This pre-mRNA would also then have a 5 prime cap added to it as well as the poly-a tail. These processes would be unaffected. C)They have to develop a way to quantify their results such as how well these digestive cells could digest certain substances and measure it over a certain amount of time. The scientists would also need a control group under the exact same environmental conditions to ensure that the drug is the factor that is changing the results between the control group and the test group. They would also need to repeat the experiment multiple times for precision and accuracy. FRQ: scientists isolated human digestive tract cells and placed them in a petri dish. To find out the significance of snRNPs in the function of a cell they then injected the cells with a drug that has been proven to prevent the snRNPs from activating. A)How would this affect the cells? B)What part of gene expression would not be affected by this drug? C) What would the scientists need to include in their experimental design for this experiment to provide accurate results

8 Learning Objective 3.21: The student can use representations to describe how gene regulation influences cell products and functions. SP 1.4: The student can use representations and models to analyze situations or solve problems qualitatively and quantitatively. Explanation: Gene regulation influences cell products and functions through processes that lead to cell specialization, transcription factors, and the use of regulatory genes. Regulatory sequences interact with regulatory proteins to control transcription through the use of promoters, terminators, and enhancers. There are also inducers that can turn on the expression of certain genes and repressors that can inhibit certain genes. Genes are inhibited through regulatory proteins binding to DNA and blocking transcription, in contrary, genes are stimulated whenever proteins bind to the DNA and stimulate transcription. To express gene regulation and how it influences cell products and functions, one can use the image above. Here we see the regulatory protein is an inhibitor and is binding to the gene sequence, inhibiting transcription. Multiple Choice Question: Choose the answer choice that places these steps in the correct order for the action of enhancers and transcription activators. I.The activators bind to certain general transcription factors and mediator proteins, helping them form an active transcription initiation complex on the promoter. II.Activator proteins bind to distal control elements grouped as an enhancer in the DNA. This enhancer has three binding sites. III.A DNA-bending protein brings the bound activators closer to the promoter. Other transcription factors, mediator proteins, and RNA polymerase are nearby. A.III, II, I B.III, I, II C.I, II, III D.II, III, I http://bio100.class.uic.edu/lectures/17_06b_gene_regulation-L.jpg FRQ: In the above image we see a cell cycle-stimulating pathway. The cell cycle stimulating pathway is one of the many signaling pathways that regulate cell division by influencing transcription. A.What is occurring within the diagram that leads to the stimulation and what is going to occur due to that stimulation? B.Explain and give an example of something that can occur due to a mutation within the cell cycle stimulating pathway. C.In contrary of the cell cycle stimulating pathway, there is the cell cycle-inhibiting pathway. What occurs within this pathway and what effect does it have on transcription? How can this be beneficial?

9 Answer Key MULTIPLE CHOICE: Choose the answer choice that places these steps in the correct order for the action of enhancers and transcription activators. I.The activators bind to certain general transcription factors and mediator proteins, helping them form an active transcription initiation complex on the promoter. II.Activator proteins bind to distal control elements grouped as an enhancer in the DNA. This enhancer has three binding sites. III.A DNA-bending protein brings the bound activators closer to the promoter. Other transcription factors, mediator proteins, and RNA polymerase are nearby. A.A is incorrect because it places the first step for the action of enhancers and transcription activators at the DNA-bending protein bringing the bound activators closer to the promoter. We need the initial binding of the activators before we can move them to the promoter. B.B is incorrect because it yet again places the first step as step III, which we have already decided can not yet occur without the activator initially binding. C.C in incorrect because although we have the initial binding of the activator proteins, we see that the second step is placed at step II, which is another binding process that is more broad than that of step I. D.D is correct because the first step is the initial binding to the distal control elements that are grouped as an enhancer within the DNA. The next step would be the DNA-bending protein bringing the bound activators closer to the promoter. The bending of the DNA by the protein allows enhancers to influence a promoter that can be hundreds of nucleotides away. The specific transcription factors, called activators, will then bind to the general transcription factors which will activate transcription initiation complex and therefore initiate the RNA synthesis. FRQ: A.There is a growth factor that initially triggers the pathway and binds to the receptor in the plasma membrane. After this step, the signal will be relay to a G protein, in this specific diagram, the G protein is identified as RAS which is active when GTP is bound to it. RAS passes the signal to protein kinases and the last kinase then activates the transcription activator which will turn on one or more genes for proteins that stimulate the cell cycle. Due to this stimulation, the cell cycle will occur more rapidly and will influence transcription. Transcription is when a particular segments of DNA is copied into RNA by RNA polymerase. B.Should there be a mutation within the cell cycle- stimulating pathway, it could result in cancer. If a mutation were to make RAS or any other pathway component super active, then cancer could occur due to excessive cell division. This would lead to an incredible amount of cells and because this cycle is stimulating, there would be nothing to tell or signal the cells to stop producing. The mutation would cause an increase in the cell production and would not allow for the cells to be turned off or to stop being produced. C.In the cell-cycle inhibiting pathway, the DNA damage is an intracellular signal that is passed by protein kinases and leads to the activation of P53. P53 promotes transcription of the gene for a protein that would inhibit the cell cycle. This results in the damaged DNA not being replicated because of the suppression of cell division. This inhibiting cycle would simply not allow transcription to occur, this can be beneficial should there be a cell that contains a disease that does not need to be copied or a mutation that does not need to be transcribed.

10 LO 4.15: The student is able to justify the selection of geological, physical, and chemical data that reveal early Earth conditions. SP 1.3: The student can justify the selection of the kind of data needed to answer a particular scientific question. Explanation: The conditions of early Earth can be determined from a variety of ways. Foremost, the famous Miller-Urey experiment demonstrated how organic compounds could be formed by simulating what earth’s atmosphere would have been like at the time. They used H2O, H2, NH3, CH4, and water vapor, and added electrical discharges to simulate the lightning that would have occurred in primitive earth, to a closed system. Upon sampling material from this system after allowing it to circulate, they found that their experiment had produced amino acids, which are the building blocks of life. They also found other organic molecules and hydrocarbons. This proved the hypothesis that early Earth could have had a reducing atmosphere. Fossils and rocks, which can be dated using radiometric dating, indicate at what point in time certain organisms existed, and also where they existed. These fossils have helped determine a geologic record, which breaks Earth’s history up into different eras, separated by times of mass extinctions. Geological evidence of fossil distribution suggests movement of the continents, or continental drift, occurring. This data suggests that the continents were connected at one point, forming a supercontinent called Pangaea. Specifically, the similarity of the fossils found in South America and Africa support the idea that the land masses were once attached. MC Question: Stromatolites are the oldest fossils found on Earth. They are made up of layers of bacteria and sediment and originally formed around 3.5 billion years ago, after earth cooled enough so that liquid water could exist. Their existence peaked around 1.25 billion years ago, around the occurrence of the Cambrian substrate revolution, which saw the rise of animals that grazed on and burrowed through bacterial mats such as stromatolites. Where might you find stromatolites today? a) On dry land, with not many other organisms around. b) In regular, warm, shallow water. c) In hypersaline, warm, shallow water. d) In regular, cold, deep water. FRQ: Millions of years of Pangaea’s drifting apart has had a drastic impact on the organisms that have once inhabited it, and now inhabit its separate land masses. a)What type of speciation occurred within the populations that were split in two by continental drift? How can this be proven? b)What other factors associated with the land movement could have caused species form and also die off? c)How could the theory of continental drift serve to explain the contrast of Australian marsupials with eutherians seen in the rest of the world?

11 MC Question: Stromatolites are the oldest fossils found on Earth. They are made up of layers of bacteria and sediment and originaly formed around 3.5 billion years ago, after earth cooled enough so that liquid water could exist. Their existence peaked around 1.25 billion years ago, around the occurrence of the Cambrian substrate revolution, which saw the rise of animals that grazed on and burrowed through bacterial mats such as stromatolites. Where might you find stromatolites today? a) On dry land, with not many other organisms around. b) In regular, warm, shallow water. c) In hypersaline, warm, shallow water. d) In regular, cold, deep water. Stromatolites today can only exist in hypersaline water, because other animals cannot. Their numbers began dwindling when grazing/burrowing animals began to consume and destroy them, so today they can only live where there are no such organisms to fall prey to. They live in warmer water because it is ideal for bacteria formation. FRQ: Millions of years of Pangaea’s drifting has had a drastic impact on the organisms that have once inhabited it, and now inhabit its separate land masses. a)What type of speciation occurred within the populations that were split in two by continental drift? How can this be proven? b)What other factors associated with the land movement could have caused species form and also die off? c)How could the theory of continental drift serve to explain the contrast of Australian marsupials with eutherians seen in the rest of the world? A) Allopatric speciation occurred because the species became physically isolated from one another. This can be seen in the similarity of the fossils found in South America and in Africa, in places where the two land masses would have been conjoined. B) Other factors as a result of continental drift influencing populations would include the deepening of ocean basins after Pangaea formed, killing off the many populations that thrived in shallow waters. When Pangaea was formed, the interior of the continent was cold, dry, and inhospitable to its inhabitants. Pangaea’s drifting together and apart over millions of years has reshaped diversity and provided opportunities for new organisms to survive while others went extinct. C) The marsupials in Australia fill roles analogous to those of mammals in other continents. Marsupials evolved on Pangaea while it was still one landmass, and when the landmass broke up, the area most of the marsupials lived on became separated from the landmass. The mammals living on the new Australia could not compete with the marsupials already there, and the marsupials left on the other landmass could not compete with the more abundant mammals, so each died off on their less inhabited continents.

12 LO 1.12: The student is able to connect scientific evidence from many scientific disciplines to support the modern concept of evolution. SP 7.1: The student can connect phenomena and models across spatial and temporal scales. Explanation: Many different sciences are used in the study of evolution. The biggest four are chemistry, geology, geography, and mathematics. Paleontologists have found fossils and have done intensive studies on the remnants of early life. These fossils are the keys to finding the slight changes in species over time. This is where chemistry comes in, it can be used for carbon dating which measures the amount of radioactive carbon remaining in the fossil. This can help in the understanding of which fossil came before another and where all of the pieces and separate species are connected. Chemistry also can show similarities in DNA and the proteins for which certain genes code. Similarities in DNA and proteins suggest common ancestry. Where a fossil is found can also show us a lot about early life and evolution and how different species came to be. This can also allow us to understand the origins of life and where the rise of different species took place. The research of the homologous structures of the genetic materials of all different type of organisms whether it be bacterium, plants, or animals, all living things have the same basic units of nucleotides. All living things also require the same basic units of life to survive which shows a common ancestor between every living thing. Vestigial structured are remnants of structures that were useful in the past but were selected against during natural selection. These structures aid in the understanding of how organisms branch off from each other and where two species may have a common ancestor. Multiple Choice Question: Recently fossils were found that suggest that early man rose in the Western region of Africa around 1.8 million years ago. What is a study that can be done to confirm this assumption? (A)Phosphorous dating (B)Gel Electrophoresis (C)DNA Replication (D)Carbon dating H. sapiensC. carettaV. indicusG.gallusG. australis Hinged Jaw XXXXX Bony Skeleton XXXX Four Walking Limbs XXX Amniotic Egg XX Fur/HairX Chart depicting phylogenetic information that can be used in creating a phylogenetic tree that depicts in which order certain species rose. Free-Response Question: The chart above depicts five different species and their homologous structures. The X in a box indicates that the structure is present in the organism. A. Create a phylogenetic tree depicting the above information. B. Explain and justify the placement of each of the organisms on your tree. http://www.bio.miami.edu/dana/pix/homologous_forelimbs.jpg This shows homologous structures between different species showing that there is a common ancestor.

13 Answer Key: Multiple Choice: Recently fossils were found that suggest that early man rose in the Western region of Africa around 1.8 million years ago. What is a study that can be done to confirm this assumption? (A)Phosphorous dating (B)Gel Electrophoresis (C)DNA Replication (D)Carbon dating Eplanation:The answer is D because carbon dating is the only one of the answer choices that makes sense on this. The carbon dating is the technique used to date radioactive carbon and this was the best way to know what order the species come in. Free-Response Question: Out of 10 points The chart above depicts five different species and their homologous structures. The X in a box indicates that the structure is present in the organism. A. Draw a phylogenetic tree depicting the above information. B. Explain and justify the placement of each of the organisms on your tree. H. sapiens C. caretta V. indicus G. gallus G. australis B.(8 points) Student must use at least one of the following terms in their response. One point rewarded for the use of each term. homologous structures similar structures common ancestor Student must justify the placement of all of the organisms correctly as it is seen on their phylogenetic tree. Extra points rewarded for examples and use of extra terms. A. (2 points) All of the organisms must be in the correct space to receive credit.

14 Learning objective 1.18: The student is able to evaluate evidence provided by a data set in conjunction with a phylogenetic tree or simple cladogram to determine evolutionary history and speciation. SP 5.3: The student can evaluate the evidence provided by data sets in relation to a particular scientific question Explanation: The cladogram to the right illustrates the evolutionary history and development of plants based on certain genetic adaptations. The diagram begins with green algae, showing that green algae is a common ancestor of all the categories of plan species further along the graph. From there, various evolutions produced new successive categories of plants, each possessing all the adaptations of the prior categories but introducing a new adaptation in the process. For example, mosses and hornworts evolved from liverworts after developing apical growth, while gymnosperms evolved from ferns after developing seeds. However, gymnosperms, as illustrated by being placed further along the graph and possessing numerous more adaptations, are more distantly related to liverworts than mosses and hornworts are, which posses only one more adaptation and are thus placed closer to liverworts on the cladogram. Multiple Choice: What can be determined from the following graph about the genetic relation of species A and E? a)Species E is the ancestor of species A, because it is lower down on the graph b)Species A is the ancestor of species E, because it is higher up on the graph c)Species A and E and the most closely related species on the graph d)Species and E are the least related species on the graph SpeciesABCD A-1515 B-312 C-9 D- The chart above shows differences in nucleotides for four different species of finches which all derived from a common ancestor. Based on this information: a)Draw a phylogenetic tree to represent the data b)Describe two additional pieces of information from these organisms that could be used to create a phylogenetic tree c)Discuss two possible causes of speciation among the finches https://en.wikipedia.org/wiki/Evolutiona ry_history_of_plants#/media/File:Plant_ Diversity_(2).svg

15 Answer Key The correct answer is D because the cladogram organizes closer together or farther apart based on how related they are, and species “A” and “E” are the two farthest apart species on the graph, they show the least evolutionary correlation and the least relation of any of the five species. Multiple Choice: What can be determined from the following graph about the genetic relation of species A and E? a)Species E is the ancestor of species A, because it is lower down on the graph b)Species A is the ancestor of species E, because it is higher up on the graph c)Species A and E and the most closely related species on the graph d)Species and E are the least related species on the graph The chart above shows differences in nucleotides for four different species of finches which all derived from a common ancestor. Based on this information: a)Draw a phylogenetic tree to represent the data b)Describe two additional pieces of information from these organisms that could be used to create a phylogenetic tree c)Discuss two possible causes of speciation among the finches a)The phylogenetic tree should show species “A” and “B” closest together on the same branch, with species “C” close to them but on a different branch, and “D” far off from all three on its own branch entirely. b)One piece of additional information could be observed physiological differences between the finch species, including the presence of vestigial structures. Another piece of additional information could be the data of the finch species’ reproduction and development during the embryonic stage including which genes are expressed and regulated. c)One cause of speciation could be geographic isolation, such as an ocean or river, that separates finch populations which then undergo divergent evolution to adapt to their specific habitats. Another cause of speciation could be a gene mutation in certain finches which cause them to experience reproductive isolation from the rest of the finch population, splitting the population into two distinct gene pools which mate and evolve separately.

16 LO 1.25: The student is able to describe a model that represents evolution within a population. SP 1.2: The student can describe representations and models of natural or man-made phenomena and systems in the domain. Explanation: Evolution acts on a population through the use of natural selection. Natural selection weeds out the less fit individuals through not letting them pass on their genes through reproduction (either by death, or infertility). This keeps the gene pool from accumulating “non-worthy” genes. If, by chance, an individual in a population gets a beneficial mutation in its genes, or an organism that immigrated to the population passes on a gene, that makes the subsequent generations more fit to survive in the environment, these genes will survive and, over a long period of time, the individuals the original organism spawned will have survived and reproduced better than the one’s that have no mutation, and the population’s gene pool will have been altered, and thus, the population will have evolved. MC Question: Which of the following describes a population that has evolved? a) A bird is born that’s beak lets it recover berries better than any other, but it dies before it can reproduce. b) A squirrel with a darker coat color than all other squirrels immigrates into a population where darker color is selected for, but it is a different species and produces sterile offspring with the native population. c) A fish immigrates into another population with the same genotype and produces viable offspring who pass down its genes over years and years. d) A dog population frequently hunts fish in a nearby lake. One dog is born with webbed feet, and passes these genes on down the line. Now most dogs in this population have webbed feet. FRQ: Define and provide a plausible example of evolution. Paul Wagner http://www.biology- online.org/images/darwin_finches.jpg

17 Answer Key MC Question: Which of the following describes a population that has evolved? a) A bird is born that’s beak lets it recover berries better than any other, but it dies before it can reproduce. b) A squirrel with a darker coat color than all other squirrels immigrates into a population where darker color is selected for, but it is a different species and produces sterile offspring with the native population. c) A fish immigrates into another population with the same genotype and produces viable offspring who pass down its genes over years and years. d) A dog population frequently hunts fish in a nearby lake. One dog is born with webbed feet, and passes these genes on down the line. Now most dogs in this population have webbed feet. Explanation: Each choice, excluding choice d, the gene pool does not change because either the mutant does not survive because of some means, it cannot produce viable offspring with the native population, or the immigrant has the same genotype as the natives. Choice d is the only choice where the gene pool is changed because of reproduction of a mutant gene carrier over time. FRQ: Define and provide a plausible and detailed example of evolution. Evolution is the change of heritable traits within a population over successive generations. As in the Galapagos islands, the populations of finches evolved by way of allopatric speciation. A population of finches came to the islands and were the foundation population. Over time, groups of sparrows traveled to different islands in the island chain. Within these different habitats, there were different environmental pressures such as living space and food type. With these pressures acting on the finches, different heritable traits, such as beak size and shape as well as bird size, were either selected for or against, and, over many generations, this caused the gene pool to change to include these specific traits giving rise to new species of finches.

18 LO 2.43: The student is able to connect the concept of cell communication to the functioning of the immune system. SP 7.2: The student can connect concepts in and across domain(s) to generalize or extrapolate in and/or across enduring understandings and/or big ideas. EK 2.D.4: Plants and animals have a variety of chemical defenses against infections that affect dynamic homeostasis. Explanation: Acquired immunity uses cell communication to defend against any foreign molecules. In acquired immune response, antigens activate Helper T cell, which gives rise to B cell and Cytotoxic T cell. B cells gives rise to plasma cells and memory B cells. Helper T cell gives rise to active and memory helper T cells. Cytotoxic T cell gives rise to memory cytotoxic T cells and activates cytotoxic T cells. Helper T cells are critical in humoral and cell- mediated immune responses. After a dendritic cell degrades a bacterium, it displays peptide antigen with a class II MHC molecule on the surface. Helper T cell’s protein, CD4, binds to a class II MHC molecule’s TCR, which promotes secretion of cytokines and signals by the dendritic cell. Activated T cell that is stimulated by cytokines gives rise to Cytotoxic T cell, which attacks on infected cell, and B cells that play a critical role in humoral immunity. B cells that is activated by T cell with the aid of cytokines differentiates into memory B cells and antibody-secreting plasma cells. Memory B cells continue to circulate in body until activated by exposure to the same antigen that triggered its formation.

19 MC Question Which of the following statements are false about cytokines? a)Memory B cells will not be made if cytokines are not released. b)Cytokines are cell signaling molecules that aid cell-to-cell communication in immune responses. c)Cytokines are protein molecules and important factors in the cell-to-cell communication. d)Cytokines released by innate immune cells do not affect inflammatory response since the response is not an acquired immunity. FRQ Question Cytokines are protein molecules that play a critical role in immune response and communicate through intercellular communication. a)Briefly explain the reception stage of cell communication b)Explain what will happen to cell-mediated immunity if cytokines are not excreted by the interaction between helper T cell and Class II MHC molecule. c)Explain how secretion of cytokines lead to a quicker response following a second exposure to the same pathogen

20 MC Answer/ Explanation Answer: d. Cytokines affect inflammatory response. Cytokines that are produced by activated macrophages regulate inflammatory reactions as they communicate with nearby cells. Inflammatory responses, such as engorged capillaries and fever, help deliver antimicrobial proteins and clotting elements to the injured area. Some cytokines are also responsible for communication between white blood cells. FRQ Answers Cytokines are protein molecules that play a critical role in immune response, and they communicate through intercellular communication. a)Briefly explain the reception stage of cell communication Signal molecule: cytokines behave as a ligand that binds to specific membrane bound receptor. The receptor alters its shape giving rise to a signal response. b) Explain what will happen to cell-mediated immunity if cytokines are not excreted by the interaction between helper T cell and Class II MHC molecule. Interaction between helper T cell and Class II MHC molecule promotes secretion of cytokines by the dendritic cell. Stimulated cytokines give rise to proliferation of the T cell, which activates cytotoxic T cell. Cytotoxic T cell responses to infected cells and cancer cells. If cytokines are not excreted, cell- mediated immunity will not work properly, and the body will limit the ability to fight against infected cells. c)Explain how secretion of cytokines leads to a quicker response following a second exposure to the same pathogen Secretion of cytokines stimulates B cell, which produce antibodies, and clones that eventually become effector cells and memory cells. Memory cells allow immune system to rapidly respond to any exposure to the same antigen.

21 LO 4.20: The student is able to explain how the distribution of ecosystems changes over time by identifying large-scale events that have resulted in these changes in the past. SP 6.3: The student can articulate the reasons that scientific explanations and theories are refined or replaced Explanation: An ecosystem is a geographic area where plants, animals, other organisms, land, and weather all work together. There are many factors such as abiotic and biotic factors that can affect the ecosystem and it’s distribution which is the area that it covers. Abiotic factors affect the organisms directly and are physical/non living chemical parts of the environment that affect the function of the ecosystem and its inhabitants. Examples of abiotic factors are temperature, rain, wind, pH, types of natural disasters, etc. Biotic factors are any living thing that affects another organism which includes human influence. All biotic factors require energy to do work and food for growth. Producers (organisms that create their own energy through biochemical processes), consumers (receive energy by consuming others), and decomposers (organisms that break down the organic matter in the dead bodies of plants and animals) are all parts of biotic factors that affect the ecosystem. Some examples include any organic matter, plants, animals, bacteria, human influence, or other things that can be found in the ecosystem. M.C Question: Which of the following is a biotic factor that affects the ecosystem? a)Humans logging an entire rainforest b)An earthquake that destroys parts of a mountainous region c) A fungus or moss that grows on trees d)A primary producer such as a plant FRQ: Consider an ecosystem and all its factors. a)If there’s 3000 J in the first trophic level how much energy is remaining by the third? b)Explain how an erupted volcano would affect an ecosystem. c)What would grow back first and why?

22 Answer Key: Multiple Choice a)No, because humans influence other organisms which is the qualification of an abiotic factor b)Yes, because an earthquake is a natural disaster which is a non living process (biotic factor) c)No, because fungus is a living organism that produces its own energy through biochemical processes which means it’s an abiotic factor d)No, because primary producers affect other organisms which makes it an abiotic factor This is correct because biotic factors are non living things that affect the ecosystem. It couldn’t have been choice a because human influence is part of an abiotic factor since one organism is influencing another. Choice c is incorrect because fungus and moss are both living organisms that provide energy through biochemical processes and choice d is incorrect because primary producers influence other organisms. Biotic and abiotic factors are very different factors but biotic factors are dependent on abiotic factors. This is because non-living variables cant affect an ecosystem is there are no living factors to disrupt or affect. FRQ: a)3,000 J of energy will decrease by ten percent for each trophic level b) An erupting volcano puts off a lot of ash in the atmosphere and lava throughout the land. The lava will destroy most if not all of the soil, plant, and animal life. If there are no primary producers such as plant life, then the other organisms that need their food/energy supply wont survive. There could also become an overpopulated amount of organisms if their predators don’t return at the same time. For example, if a lot of birds are killed then there will be a vast amount of insects. c ) The first organism to come back is lichen because they can live without and will cling directly to the rock created by the volcanic eruption. These organisms will begin to break down the rock and will dry out or decompose if there’s not enough rain. Since the lichen and rock have broken down the wind will cause the dust particles to fall between cracks of the rock or stone. This will cause small patches of soil to form. The wind will then also be able to blow seeds into the cracks/soil, they’ll germinate, and new plant life will begin to form. This process will repeat a vast amount of times and eventually the plant life will flourish like normal.

23 Learning Objective 1.24: The student is able to describe speciation in an isolated population and connect it to change in gene frequency, change in environment, natural selection, and/or genetic drift. Science Practice 7.2: The student can understand the situation given and predict the results related to the learning objective. Explanation: When there is a species in an isolated population there tends to be changes within that species. One of them is gene frequency which is the ratio of a particular allele to a total of all other alleles of the same gene in a given population. A change in the environment also causes a change in gene frequency by initiating adaptation. Natural selection is differential success in the reproduction of different phenotypes resulting from the interaction of organisms with their environment, therefore, evolution occurs. Genetic drift is also a major component in speciation because it is the unpredictable fluctuations in allele frequencies from one generation to the next because of a population’s finite size. MC: Koalas, unlike other animals, have opposable thumbs and long nails to be able to climb trees. Also, on the hind paw there is no claw on the biggest digit, which is opposable to the others for gripping. If their habitat was destroyed and there were no trees left, what would happen to the characteristics of the koala overtime in future generations? a.They would lose their fur. b.Their paws would shrink. c.They would lose their opposable digits. d.They would gain a massive amount of weight due to lack of exercise (climbing trees). FRQ: Explain how each of the three factors shown in the picture cause speciation to occur within generations of bunnies. What could be the reason for the change in fur color of these bunnies?

24 Multiple Choice answer key: Koalas, unlike other animals, have opposable thumbs and long nails to be able to climb trees. Also, on the hind paw there is no claw on the biggest digit, which is opposable to the others for gripping. If their habitat was destroyed and there were no trees left, what would happen to the characteristics of the koala overtime in the future generations? a.They will lose their fur. b.Their paws would shrink. c.They will lose their opposable digits. d.They would gain weight from lack of exercise (climbing trees). Explanation: If there are no trees for the Koalas to climb then they will have to adapt to staying on ground and their opposable digits make it uncomfortable so overtime, since those digits will no longer be of use, they will start to decrease in koalas and offsprings will have shorter and shorter opposable digits until they are finally gone. FRQ answer key: Explain how each of the three factors shown in the picture cause speciation to occur within generations of bunnies. What could be the reason for the change in fur color of these bunnies? 1.Geographic isolation of populations: Depending on where the bunnies are located, the environment will have many different changes such as temperature, weather, and vegetation. If it is too hot their genes might make them have less fur to keep them somewhat cool. In this case, the bunnies might have gone from being in a warm climate to a much frigid climate, therefore, the fur of the bunny got darker to attract more light/heat. 2.Mutations and natural selections: Mutations can also change the physical characteristics of animals but so can natural selection. An example of how it works is the animal’s body changing to survive and produce more offsprings. White furred bunnies are more noticeable when against a dark colored object, therefore, are an easier target to spot. Natural selection occurs by changing the color of the bunny to a darker color so it can blend in. 3.Reproductive isolation produces two species: Reproductive isolation refers to the situation where different species may live in the same area, but properties of individuals prevent them from interbreeding, which leads to the production of two different species.

25 LO 4.5: The student is able to construct explanations based on scientific evidence as to how interactions of subcellular structures provide essential functions. SP 6.2: The student can construct explanations of phenomena based on evidence produced through scientific practices. Explanation: The differences between prokaryotic and eukaryotic cells mainly comes down to subcellular structures. Prokaryotic cells contain no true nucleus or membrane bound organelles while eukaryotic cells contain both. In an eukaryotic cell there are many different membrane bound organelles each with a different function that the cell need for survival. Each function of each organelle works with the others to create what the cell needs to survive; the organelles are codependent of each other. Many of the organelles of a cell are part of the Endomembrane System which is a collection of membrane bound organelles inside an eukaryotic cell related through direct contact or by transfer vesicles. These organelles are the nuclear envelope, endoplasmic reticulum, golgi apparatus, lyosomes, vacucoles, and the plasma membrane. The organelles that aren’t part of the endomembrane system but still crucial to the survival of the cell are the mitochondria, chloroplast (found in plant cells only) and peroxisomes. MC Question: If a mutation were to occur in the nucleus that resulted in a lack of protein synthesis, what did the mutation most likely effect? A)nuclear lamina B)Ribosome production C)nucleolus D) nuclear envelope FRQ: When indentifying a cell, the first thing that scientists look for is membrane bound organelles. a.Identify three different organelles and their functions that would be found in an eukaryotic cell. b.Identify two different subcellular structures that would classify the cell as a plant cell.

26 MC Question: If a mutation were to occur in the nucleus that resulted in a lack of protein synthesis, what did the mutation most likely effect? A)nuclear lamina B)Ribosome production C)nucleolus D) nuclear envelope Explanation: Ribosomes are made in the nucleus and are also responsible for the synthesis of proteins. If a mutation were to occur in the nucleus that reduced protein synthesis then the mutation effected the production of ribosomes. FRQ: When indentifying a cell, the first thing that scientists look for is membrane bound organelles. a.Identify three different organelles and their functions that would be found in an eukaryotic cell. b.Identify two different subcellular structures that would classify the cell as a plant cell. a.The nucleus is found in eukaryotic cells which is where the genetic material is found in the cell. It is also where ribosomes are made. Mitochondria are also found in an eukaryotic cell, and is the site of cellular respiration which generates ATP. The golgi appartatus is the center of manufacturing, warehousing, and shipping. This means that the ER products, such as membrane phospholipids and sugars of glycoproteins, get modified and sent along to other organelles. b.Chloroplast, which is where photosynthesis occurs, is only found in plant cells. Central vacuole is also a structure that is only found in plant cells. If either of these structures were found in a cell then the cell would be identified as a plant cell.

27 LO 3.28 The student is able to construct an explanation of the multiple processes that increase variation within a population. SP 6.2:The student can construct explanations of phenomena based on evidence produced through scientific practices. Explanation: Variation within a population, specifically genetic variation, can be the result of random mating, random fertilization, recombination between homologous chromosomes during meiosis (which mixes up the alleles within an organism’s offspring), and mutation (which can add entirely new alleles to a population). Genetic variation is a significant contributor to evolution because it allows the increase or decrease in frequency of alleles present within a population. Random mating is the ability of individuals to move and breed freely with other individuals in a non-fixed pattern (regardless of any preferences) and thus allows for a new combination of alleles in the offspring’s generation. Fertilization in its natural form is completely random because of the miniscule likelihood that one specific sperm with fertilize another specific egg. This process allows there to be an almost infinite amount of variation that can occur when fertilization is practiced. Recombination is the process in which an offspring is created that has a blend of traits unique from both its parents due to the exchange between homologous chromosomes that leads to variety in the population. The essential role of these processes to variation within a population can be seen in scientific experiments where conditions and behaviors are specifically altered by scientists to create populations of genetic homogeneity. Scientific practices to identify genetic variation can include the study of phenotypic variation in quantitative or discontinuous traits. MC Question: A mad scientist named Jimmy Neutron has discovered a way to completely eradicate all types of mutations in DNA sequences within the Drosophila species. How will this ultimately effect the Drosophila population moving forward? A)This will have no effect on the population. B)Because mutations have only harmful effects on organisms this discovery will save the future generations of this species. C)The population will have greater variation because almost all individuals will survive to reproduce. D)Evolutionary change will decrease because mutations are an essential mechanism to variation within a population. FRQ: Jimmy Neutron is at it again! He has begun an unauthorized experimental study where he specifically pairs up couples of similar phenotypic characteristics to breed within a population of 100 moths. He breeds five generations of moths in this manner. a. What natural mechanism has Jimmy Neutron taken out of the picture in this experiment? Explain how this phenomena contributes to the make-up of a population. b. If Jimmy Neutron wanted to identify any mutations within individuals of the population where would he search for those? Do mutations always affect the individual’s offspring? c. Could these mutations have a positive effect on the population? If so, how? d. How does meiosis factor into changes in diversity throughout the population from generation to generation? Be specific. http://www.darwinwasright.org/genetics.html The moths portray the diversity of genetic and phenotypic variance in a population.

28 MC Question: A mad scientist named Jimmy Neutron has discovered a way to completely eradicate ALL types of mutations in DNA sequences within the Drosophila species. How will this ultimately effect the Drosophila population moving forward? A)This will have no effect on the population. B)Because mutations have only harmful effects on organisms this discovery will save the future generations of this species. C)The population will have greater variation because almost all individuals will survive to reproduce. D)Evolutionary change will decrease because mutations are significant contributors to variation within a population. Answer Key FRQ: Jimmy Neutron is at it again! He has begun an unauthorized experimental study where he specifically pairs up couples of similar phenotypic characteristics to breed within a population of 100 moths. He breeds five generations of moths in this manner. a. What natural mechanism has Jimmy Neutron taken out of the environment in this experiment? Explain how this phenomena contributes to the make-up of a population. b. If Jimmy Neutron wanted to identify any mutations within individuals of the population where would he search for those? Do mutations always affect the individual’s offspring? c. Could mutations have a positive effect on the population? If so, how? d. How does meiosis factor into changes in diversity throughout the population from generation to generation? Be specific. a.Jimmy Neutron has stripped the mechanism of random mating from the population, which is the the ability of individuals in a population to mate freely in a non specific pattern. This phenomena is known to increase variation within the make-up of the population because it allows for a greater number of possible allelic combinations. b. Mutations within individuals can be found in the nucleotide sequences of the genome of the organism. Mutations do not always affect the offspring but they can. Some mutations such as somatic mutations occur in non-reproductive cells and thus do not affect offspring. Other mutations known as germ line mutations occur in reproductive cells and are inherited by the individual’s offspring. c.Mutations can positively effect the population by adding variation within that population. Their introduction can help prevent the population from becoming completely homogeneous in nature. Some mutations are even selected for and increase reproductive success thus proliferating their existence within the gene pool. d.Recombination and independent assortment in meiosis causes each gamete to have its own distinct hereditary material or DNA. A new combination of genes is present in the resulting zygote. This leads to increased variation within offspring because they are uniquely different from both of their parents. Explanation: Mutations are errors which produce alternative genes (that may not already be in the gene pool), which could be an advantage selected for evolutionarily. Mutations can be harmful, beneficial, or neutral to organisms but when they are harmful they predominately prevent organisms from reaching sexual maturity and reproducing, so they would not be inherited by future generations. Without the beneficial mutations occurring in the population an important factor to genetic variation is inhibited. Variance in genotypes and phenotypes is most commonly introduced by the appearance of new mutations and this accumulation causes species to evolve over time. Thus when ALL mutations are inhibited evolutionary change is negatively affected.

29 LO 4.23: Student can construct explanations of the influence of the environmental factors on the phenotype of an organism. SP 6.2: The student can construct explanations of phenomena based on evidence produced through scientific practices. Explanation: The nature vs. nurture dispute is one that is widely debated among many sciences and other areas of study. The logical response concerning this argument is the assertion that a character is shaped by both environment and genotype – multifactorial characters. An important concept to comprehend is that a genotype is not usually associated with one rigidly defined phenotype, but rather with a range of phenotypic possibilities due to environmental influences such as temperature, light, predation, available resources, etc. This phenotypic range is called the norm of reaction for a genotype. Phenotypic plasticity is the change in a phenotype as the result of environmental conditions This is the reason behind why a single tree with its inherited genotype has leaves that vary in size, shape, and color. For humans, it explains how nutrition influences height, exercise alters build, sun tanning darkens skin, and experience improves performance on intelligence tests. Even identical twins, who are genetically indistinguishable, accumulate phenotypic differences as a result of different exposure to their environments. MC Question: Scientists have discovered a new life-sustaining planet called “Xenon” in a nearby galaxy. The gene structures of the organisms found on Xenon seem to be very similar to those that are present on Earth; specifically, an animal that appears to be identical to Earth’s Arctic fox. On Earth, the Arctic foxes have a brown coat during the warmer seasons, and a white coat during the snowy winter months. On Xenon, where it snows all year round, the foxes are always white. Which of the following MOST LIKELY accounts for this difference? A) The genes for both species of foxes originated on planet Earth. B) The foxes on planet Earth and planet Xenon share a common ancestor. C) The genotypes found in Xenon’s foxes happen to be identical to Earth’s foxes. D) These phenotypes are selected for or against by the environmental factors relative to each planet. E) There are more fox predators on Xenon than on Earth. FRQ: A wealthy man in Lewisville, North Carolina has decided to develop a neighborhood around a manmade, freshwater lake called Lissara. Lissara has many of the same organisms that are found in natural aquatic ecosystems. However, when comparing identical species from Lissara and the nearby Yadkin River, several phenotypic differences can be observed. a. Explain the mechanisms of individual variation in populations. b. A dam is constructed in the middle of Lake Lissara. Describe TWO phenomena of adaptive phenotypic change that may result from developmental drift. c. The houses built around the lake are very tall, and do not allow much sunlight to enter the ecosystem. A scientist compares a plant sample from Lissara to a plant grown in an environment with lots of sunlight. Describe why differences may exist between the samples. http://people.tamu.edu/~tdewitt/

30 Answer Key MC Question: Scientists have discovered a new life-sustaining planet called “Xenon” in a nearby galaxy. The gene structures of the organisms found on Xenon seem to be very similar to those that are present on Earth; specifically, an animal that appears almost identical to Earth’s Arctic fox. On Earth, the Arctic foxes have a brown coat during the warmer seasons, and a white coat during the snowy winter months. On Xenon, where it snows all year round, the foxes are always white. Which of the following MOST LIKELY accounts for this difference? A) The genes for both species of foxes originated on planet Earth. B) The foxes on planet Earth and planet Xenon share a common ancestor. C) The genotypes found in Xenon’s foxes happen to be identical to Earth’s foxes. D) These phenotypes are selected for or against by the environmental factors relative to each planet. E) There are more fox predators on Xenon than on Earth. Explanation: Because environmental factors are likely to differ from one place to another, natural selection can contribute to geographic variation. Most species exhibit this, as differences between the gene pools of separate populations and subgroups are observed. Many animals have developed special adaptations that let them change their coloration as their surroundings change. An example of a specific environmental factor that determines this shift is the changing of season. In the spring and the summer, a mammal’s habitat may be full of greens and browns, while in the fall and winter more snow is present. Animals that have the selective phenotypic advantage of camouflage will be able to adjust to their environment to stay concealed from predators. FRQ: A wealthy man in Lewisville, North Carolina has decided to develop a neighborhood around a manmade, freshwater lake called Lissara. Lissara has many of the same organisms that are found in natural aquatic ecosystems. However, when comparing identical species from Lissara and the nearby Yadkin River, several phenotypic differences can be observed. a. Explain the mechanisms of individual variation in populations. b. A dam is constructed in the middle of Lake Lissara. Describe TWO phenomena of adaptive phenotypic change that may result from developmental drift. c. The houses built around the lake are very tall, and do not allow much sunlight to enter the ecosystem. A scientist compares a plant sample from Lissara to a plant grown in an environment with lots of sunlight. Describe why differences may exist between the samples. a. Individual variation occurs in populations of all species. These variations are always present, and heredity is undoubtedly a raw mechanism for natural selection. However, not all variation can be credited to heritability. Phenotype is the cumulative product of an inherited genotype and a multitude of environmental influences. When individuals noticeably represent two or more distinct characteristics are represented in high concentrations, a population is said to display phenotypic polymorphism b. Fluctuations in how a genotype is physically expressed come from a mechanism called genetic drift. Two situations that can increase the likelihood of genetic drift and in turn, largely impact the environment are referred to the bottleneck and the founder effect. A sudden change in the environment, such as fire or flood may drastically reduce the size of a population. The survivors have passed through a restrictive “bottleneck”, and their gene pool may no longer be reflective of the original population’s gene pool. Certain phenotypes may be overrepresented while others may be underrepresented, and some may be eliminated altogether. This mechanism effectively represents how environmental factors can affect an organism’s phenotype. When a few individuals are isolated from a larger population, the smaller group is likely to contract adaptations that are better fit for their selective advantage in their environment, which could potentially alters their homogeneous appearance from the original population. c. If environmental conditions become unfavorable for the survival of an organism, it normally has two options: either to move or change in a way that allows it to function under the new conditions. For plants, which are nonmotile, the only alternative is to adjust to the new conditions, usually by an alteration of growth pattern or modification of subcellular machinery. The ability of an individual to change its morphological and/or physiological features after exposure to different environmental conditions is known as phenotypic plasticity. A plant is more likely to survive and reproduce in an unstable environment if an individual can modify itself morphologically and physiologically. Since photosynthesis is the sole source of energy for most plants, light is one of the key requirements for survival and growth. Capture of the optimum amount of light, especially in the face of competition from other plants, is extremely important. Thus, it is not surprising that plants from environments with fluctuating light levels like Lissara have evolved high levels of phenotypic plasticity in response to changes in light intensity.

31 Helpful: Additional Diagrams and Figures! Figure 4.23.1: The pH of the soil will change the color of hydrangea flowers from blue to pink. 4.23.1 4.23.2 Figure 4.23.2: Plasticity of the epigenome during development affords an opportunity for the developing organism to 'pre-adapt' to the future adult environment, which provides a survival advantage. However, in settings in which the fetal environment does not match the adult environment — for example, fetal development in a nutrient-poor environment (such as maternal starvation) coupled with a nutrient-rich adult environment — the resulting 'catch-up' growth and disconnection between fetal programming and the adult environment can predispose to adult metabolic disease, including obesity and type II diabetes. http://www.nature.com/nrc/journal/v12/n7/fig_tab/nrc 3220_F1.html https://labchirps.wordpress.com/tag/phenotypic- plasticity/ 4.23.3 Figure 4.23.3: Pairs of prickly and non prickly leaves borne on contiguous nodal positions of the same branchlet for four of the heterophyllous Ilex aquifolium trees sampled for comparative DNA methylation analyses. Scale bar = 5 cm. http://digital.csic.es/bitstream/10261/74205/1/boj 12007.pdf

32 LO 3.45: The student is able to describe how nervous systems transmit information. SP 1.2: The student can describe representations and models of natural or man-made phenomena and systems in the domain. Explanation: The nervous system is a complex array of neurons and support cells (astrocytes, radial glia, oligodendrocytes, and Schwann cells) that are responsible for sensory input, integration, and motor output. In the nervous system, communication and transmission of information is achieved by signaling between neurons. As opposed to paracrine and endocrine signaling pathways that solely involve chemical messages, neuron communication involves sending of electrical impulses that cause the release of ions – potassium and sodium – that trigger an electrical impulse in the next neuron. The area where these chemicals are released and interpreted is called the synaptic cleft or gap. An individual neuron has a resting charge of -70 mV (millivolts) due to high concentrations of negatively charged ions found in DNA. The charge just outside the neuron is positive due to high concentrations of positive ions – sodium and potassium. Due to the differences in charge, a voltage gradient exists across the membrane of the neuron. When a signal is received, sodium channels - which are integral proteins – allow sodium ions to flow into the neuron, thus depolarizing the neuron. If and only if the neuron’s charge drops below -55mV the neuron fires, sending a signal down the axon to the synaptic cleft between the current neuron and the next. This conditional firing is why neurons are known to have an all or none response because the neuron either sends a signal of identical strength, or does not fire at all. When such a depolarization occurs, the rapid depolarization – from -70mV to approximately 35mV – is known as an action potential. Action potentials flow quickly down the axon (120 m/sec!!!) hopping from Schwann cell to Schwann cell onto nodes of Ranvier, which are gaps between Schwann cells where sodium and potassium channels are located. When an action potential reaches the end of a neuron’s axon, chemical neurotransmitters are released from the presynaptic membrane by an influx of calcium ions through voltage-gated channels. These calcium ions allow neurons to release neurotransmitters to adjacent neurons, so that the neurotransmitters may ligand bind to the potassium and sodium channels and open them. After an action potential is created, the neuron is repolarized by opening potassium channels to release potassium back outside the cell, recreating a charge gradient across the membrane. This repolarization is never exact, as too many potassium ions are realized, called an overshoot because the membrane becomes briefly hyperpolarized before returning to the resting potential of -70mV. It is false to believe, however, that all synaptic signals cause an action potential, as there are two kinds of synaptic signals: excitatory synapses and inhibitory synapses. Excitatory synapses depolarize a neuron and promote an action potential while inhibitory synapses hyperpolarize a neuron and prevent an action potential. When multiple excitatory and inhibitory synapses are received by a signal neuron, it is the axon hillock that determines the net charge on the neuron and thus if an action potential will occur or not. Throughout the process of neural signaling, a neuron’s charge is held at -70mV by sodium-potassium pumps that use ATP to move potassium ions into the cell and sodium ions out of the cell, thus maintaining the resting potential of -70mV. MC Question: Describe the process of a single neuron firing from reception of the signal(s) to passing the signal along to other neurons. 1.Action Potential occurs 2.Hyperpolarization of neuron 3.Sodium gates open 4.Potassium gates open 5.Net charge of neuron determined A)1, 2, 3, 4, 5 B)5, 4, 3, 1, 2 C)5, 2, 3, 4, 1 D)5, 3, 1, 4, 2 FRQ: Consider a synapse that fires such as the one in Figure 1: a.Describe the curve of voltage and time from beginning to end with a discussion of what causes each change in the curve. b.Scorpion venom is known to prolong the opening of sodium channels, propose a consequence of a human being injected with scorpion venom. c.Tetrodotoxin is a poison produced by many animals, but most commonly found in puffer fish. This toxin acts by blocking Sodium ion channels. Propose a consequence of a human being injected with Tetrodotoxin. d.Match and name the two types of synaptic signals to the two curves found in Figure 2, and state which could have effects similar to Tetrodotoxin and which could have effects similar to Scorpion Venom. http://meddic.jp/inhibitory_postsynaptic_potentials http://starklab.slu.edu/PhysioLab/ap.jpg http://hyperphysics.phy-astr.gsu.edu/hbase/biology/actpot.html Figure 1 Figure 2

33 Answer Key MC Question: Describe the process of a single neuron firing from reception of the signal(s) to passing the signal along to other neurons. 1.Action Potential occurs 2.Hyperpolarization of neuron 3.Sodium gates open 4.Potassium gates open 5.Net charge of neuron determined A)1, 2, 3, 4, 5 B)5, 4, 3, 1, 2 C)5, 2, 3, 4, 1 D)5, 3, 1, 4, 2 Explanation: Before any sort of neuron firing can occur, the axon hillock must first determine the net charge on the neuron (5). After this is done and the threshold has been crossed, sodium gates open (3) to rapidly depolarize the membrane. This creates an action potential (1), after which potassium gates are opened (4) to repolarize the membrane. This causes an overshoot, where the neuron is hyperpolarized due to over release of potassium ions (2). FRQ: Consider a synapse that fires such as the one in Figure 1: a.Describe the curve of voltage and time from beginning to end with a discussion of what causes each change in the curve. The curve begins at point 1 with the membrane potential at the resting position of -70mV. At point 2, an excitatory stimulus is received that causes sodium channels to open and rapidly depolarize the membrane. This leads to the action potential at point 3, where the membrane potential has quickly flowed from -70mV to approximately 35 mV. Following the action potential, potassium channels open to repolarize the membrane (point 4), but the channels stay open for too long, causing an overshoot of the resting potential at point 5. This process is known as hyperpolarization because a negative gradient is being reestablished across the membrane. b.Scorpion venom is known to prolong the opening of sodium channels, propose a consequence of a human being injected with scorpion venom. Keeping the sodium channels open would lead to prolonged action potentials in neurons. This would occur as sodium ions would continue to flow into the cell, since the sodium channels that allow them to flow would remain open. Such prolonged action potentials would lead to overstimulation and over- signaling in the brain, which could lead to tremors, convulsions, and eventually death. c.Tetrodotoxin is a poison produced by many animals, but most commonly found in puffer fish. This toxin acts by blocking Sodium ion channels. Propose a consequence of a human being injected with Tetrodotoxin. By blocking sodium ion channels from opening, Tetrodotoxin essentially prevents any action potentials from occurring. This lack of action potentials would prevent any messages from spreading between neurons, thus severely lowering nervous system activity. In humans, this could lead to paralysis, as neurons are unable to fire to move muscles. d.Match and name the two types of synaptic signals to the two curves found in Figure 2, and identify which could have effects similar to Tetrodotoxin and which could have effects similar to Scorpion Venom. The upper curve corresponds to an excitatory postsynaptic potential (EPSP), while the lower curve corresponds to an inhibitory postsynaptic potential (IPSP). The excitatory signal (upper curve) corresponds with the scorpion venom because both are capable of causing action potentials while the inhibitory signal (lower curve) corresponds to the tetrodotoxin because both are capable of preventing action potentials.

34 LO 3.24The student is able to predict how a change in genotype, when expresses as a phenotype, provides a variation that can be subject to natural selection. SP 6.4 The student can make claims and predictions about a natural phenomena, based on scientific theories and models SP 7.2 The student can connect concepts in and across domain(s) to generalize or extrapolate in and/or across enduring understandings and/or big ideas Explanation: Charles Darwin’s theory of natural selection is based on the idea of reproduction of the fittest. The theory claims that individuals within a varied population that have more desirable traits are more likely to survive and reproduce. When a parent possesses a trait best suited for survival in a habitat, their offspring have a chance of inheriting that genotype and either becoming a carrier of that gene or having that gene expressed as a phenotype. The passing of genes from one generation to another is ensured diversity in sexually reproducing organisms via meiosis which guarantees that each gamete receives one haploid set of chromosomes. As time passes and generations go by the population changes. As a habitat changes the most desirable traits change. Natural selection occurs and the population changes again. As a more suited trait appears in a population, the individuals who posses that trait survive and reproduce altering the future generations. MC Question: A forest fire burns all the shrubs that are the main food supply for the local herbivore population. A few of the herbivores have the ability to climb trees while some don’t. How genotype of the average individual in the population change over time? A.The population will have no genetic change. B.The genotype of the herbivores will be favored by natural selection because the main food supply is on the ground. C.The herbivore population will change so the genotype produces a majority of climbing herbivores because the y have a more desirable trait to reproduce and reach a food supply D.The herbivore population will change because natural selection favors meiosis providing each gamete with a complete set of chromosomes. FRQ: Consider Darwin’s theory of natural selection. a.Compare and contrast Darwin’s theory of natural selection with Lamarck’s idea of use and disuse in terms of gene and trait variation. b.Using the figure provided, explain how the following statements apply to the genotypes, phenotypes and inheritance within a population. - Mutations - Selective Mating - Gene flow https://qph.is.quoracdn.net/main-qimg- 9645630483efae0fabac0c411f73b5ce?convert_to_webp=true

35 Explanation: The answer is C because of the term reproduction. Natural selection concerns the reproduction of species to benefit future generations, not the survival of one individual. In order for a population to exhibit natural selection, change over time must be evident therefore the genotype of previous generations best suited to survive in that habitat must alter the phenotype of the later generations. Explanation: a.Darwin’s theory of natural selection is about the reproduction of the most fit in a population. His theory also is about descent with modification of the genotype of individuals in a population. Lamarck’s theory is based on the idea of use and disuse, and that the offspring inherit the traits their parents have acquired in their lifetime and not based on their genotype. Both theories account for the evolution of traits from generation to generation and how a species responds to changes in their environment. The theories are different in terms of time because Lamarck’s theory claims that evolution can occur in one generation while Darwin taught that it took multiple generations to see evolution in a population. b. -Mutations: Mutations are changes in the organism’s DNA to create new alleles. Mutations change the genotype of an individual and may produce a phenotype that may benefit the individual. If a mutation alters the genotype of a individual in a population and the mutation produces a phenotype that is better fit for the environment, the individual with the mutation will have a high chance of surviving and their offspring inheriting the trait and also surviving. -Selective mating is when favored alleles are selected for reproduction. This applies to natural selection because those with the favored genotypes and phenotypes create variability within a population enabling certain individuals to be better suited for surviving. This influences future generations chance of surviving because selective mating favors certain alleles that are more likely to be inherited by future generations, diversifying the next generation. -Gene flow: Gene flow is the gain of loss of alleles due to emigration and immigration. The change of alleles in a population alters the genotypes and phenotypes of the population as a whole. With the introduction of new individuals, they mate with the population creating a new genotype inheritance that changes the phenotype of a population and is now subject to natural selection. MC Question: A forest fire burns all the shrubs that are the main food supply for the local herbivore population. A few of the herbivores have the ability to climb trees while some don’t. How genotype of the average individual in the population change over time? A. The population will have no genetic change. B. The genotype of the herbivores will be favored by natural selection because the main food supply is on the ground. C. The herbivore population will change so the genotype produces a majority of climbing herbivores because the y have a more desirable trait to reproduce and reach a food supply D. The herbivore population will change because natural selection favors meiosis providing each gamete with a complete set of chromosomes. FRQ: Consider Darwin’s theory of natural selection. a. Compare and contrast Darwin’s theory of natural selection with Lamarck’s idea of use and disuse in terms of gene and trait variation. b. Using the figure provided, explain how the following statements apply to the genotypes, phenotypes and inheritance within a population. - Mutations - Selective Mating - Gene flow

36 LO 2.27 The student is able to connect differences in the environment with the evolution of homeostatic mechanisms. SP 7.1 The student can connect phenomena and models across spatial and temporal scales. Explanation Life is incredibly delicate in the sense that organisms are highly sensitive to changes within the environment; as a result, they have evolved assorted mechanisms to maintain internal balance, called homeostasis, to combat environmental pressures that may disrupt it. For example, internal heat regulation is highly important to all organisms as many processes are either dependent on heat to function or function more optimally at specific temperatures. As a consequence of varying temperatures across biomes, organisms have differing levels of fatty, insulating cells which typically increase in quantity with decreasing temperatures both due to geographic causes (i.e. elevation, latitude) and climatological sources (i.e. climate change, seasonal shifts). A real world example of this is the differing levels of fatty tissues between animals living in Polar biomes versus those of Equatorial organisms, or the differences in the levels of fatty tissues within individuals of a population over the course of a year, with them rising before the winter and dropping afterwards to balance the temperature shift. Multiple Choice When a plant is exposed to an environment that has increased levels of water surrounding the roots, the number of stomata on the leaves which are open increases. Why would this be considered a homeostatic mechanism? A) The increased number of open stomata releases more water from the plant to compensate for the increased amount of water taken in to maintain typical water potential and water levels. B) By allowing a greater amount of CO 2 into the plant, it compensates for the increased amount of photosynthesis due to higher amounts of water available for reduction. C) Decreasing the number of closed stomata would promote the ability of the plant to regulate the amount of sucrose flowing through the phloem. D) Changing the amount of open stomata has no effect on homeostasis as it simply allows for more O 2 to leave the plant. Harper Day AP Biology Period 1 https://mountainhightree.com/2013/09/ the-essential-nutrient-carbon/ FRQ A population of tropical finches finds itself amidst a massive climate shift causing a drastic drop in the overall temperature of the environment. 1)a)) Explain the effects of climate change on a general environment. b)) Describe the effects of the shift on the population of finches. 2)What would be a likely outcome if the finches existed within a temperate climate prior to the climate shift? 3)Propose a change that might occur within the biotic factors of the ecosystem of the finches other than within the finches themselves.

37 Answers Multiple Choice When a plant is exposed to an environment that has increased levels of water surrounding the roots, the number of stomata on the leaves which are open increases. Why would this be considered a homeostatic mechanism? A) The increased number of open stomata releases more water from the plant to compensate for the increased amount of water taken in to maintain typical water potential and water levels. B) By allowing a greater amount of CO 2 into the plant, it compensates for the increased amount of photosynthesis due to higher amounts of water available for reduction. C) Decreasing the number of closed stomata would promote the ability of the plant to regulate the amount of sucrose flowing through the phloem. D) Changing the amount of open stomata has no effect on homeostasis as it simply allows for more O 2 to leave the plant. Reasoning The purpose of opening the stomata within a plant is to promote gas exchange with the environment, taking in carbon dioxide and releasing water and oxygen. By releasing water, the plant reduces its own water potential, increasing the uptake of water by the roots. When there is an excess of water, the plant absorbs more of it, resulting in an overall decreased water potential, meaning that it must release water to return to homeostasis. FRQ A population of tropical finches finds itself amidst a massive climate shift causing a drastic drop in the overall temperature of the environment. 1)a)) Explain the effects of climate change on a general environment. b)) Describe the effects of the shift on the population of finches. 2)What would be a likely outcome if the finches existed within a temperate climate prior to the climate shift? 3)Propose a change that might occur within the biotic factors of the ecosystem of the finches other than within the finches themselves. Answers 1a) Overall, climate change results in the general disruption of the homeostasis of the majority of organisms by changing the typical environmental pressures from what the existing species are adapted to. 1b) The decrease in temperature would result in an increase of both fatty tissue buildup and metabolism within individuals so that internal heat homeostasis could be maintained. Some individuals may die due to their inability to adjust to new conditions. 2)Had the finches lived within a cooler, temperate climate, few finches may die and they would be better suited to colder temperatures, but the general shift would be about the same. 3)As a result of decreasing temperatures, the majority of plant species would suffer as their metabolic processes slow down as proteins no longer function optimally; other organisms may suffer a similar fate as the finches, but the majority of cases would be within warm-blooded animals. Harper Day AP Biology Period 1

38 L.O. 2.15: The student can justify a claim made about the effects on a biological system at the molecular, physiological, or organismal level when given a scenario in which one or more components within a negative regulatory system is altered. S. P. 6.1 The student can justify claims with evidence. Explanation: There is a set point within the internal environment of any organism that has to be kept within a certain range for that organism to sustain life. Negative feed back adjusts certain physiological processes returning the internal environment to the range of the set point. Alterations in these mechanisms often result in very detrimental consequences. MC: An organism has several ways of detecting change within itself and outside. Thermoreceptors are one of them. The hypothalamus and the skin both act as receptors for temperature fluctuation. The hypothalamus sends impulses through the body causing dilation of the veins and increased sweat production to reduce the temperature of the organism. Which mechanism is being used to regulate the body with the external changing stimuli? A. Complement cascade B. Negative feedback. C. Interferons D. Operant Conditioning (http://media.opencurriculum.org/articles_manual/ck12_biology/homeostasis-and-regulation-in- the-human-body/1.png) FRQ: Homeostatic regulation within the body is necessary for survival. There is a very limited range that any organism can fluctuate within. Temperature regulation being the most common. If the temperature drops, the body begins to shiver to increase the temperature. If an organism is hungry, the metabolism slows allowing the organism to survive with less food than normal. A. Describe all the individual steps of the process required to regulate body temperature. Make sure to include all the organs and glands included. B. Explain what happens during blood glucose regulation if the negative feedback mechanism is interrupted.

39 MQ Answer: A. no, because this is a biochemical process in the blood that helps the immune system eliminate invading pathogens. B. Yes. C. no, because this is a protein released by animal cells that has the property of inhibiting virus replication. D. No, because this is a type of learning done vouluntarily. FRQ A.Sensors in the central nervous system send signals to the hypothalamus telling the body that the internal temperature is increasing. The brain then sends impulses to other organs, such as the skin and the blood vessel triggering a response to the increase in body temperature. Vessels begin to dilate and sebaceous glands release sweat cooling the outside of the body. B.Normally, if the body has an increase in its blood glucose level the pancreas receives a signal to release insulin for the break down of the glucose. If some part of this mechanism is inhibited then there will be a build up of glucose resulting in diabetes.

40 A.Learning Objective: The student is able to describe representations and models illustrating how genetic information is translated into polypeptides. B.Science Practice: The student can describe the representation and models of natural or man-made phenomena and systems in the domain. C.Statement : In order to express the genetic information and direction contained in DNA, the process of transcription and translation from DNA to RNA to mRNA to tRNA to amino acids to polypeptide must occur. In a eukaryotic cells the genetic information is transcribed in the nucleus and can only leave when the mRNA is mature. It must be transcribed from DNA to RNA because protein language only contains Uracil, Adenine, Guanine, and Cytosine, where as DNA contains Thymine instead of Uracil, so it must be transcribed to RNA. Translation occurs in the cytoplasm at the ribosome. However, in prokaryotic cells transcription and translation occurs in the cytoplasm simultaneously. In accomplishing Learning Objective 3.4 the student will be able to explain how the body translates genetic information, stored as DNA, into proteins that allows the body to direct cellular process including; expressing phenotypes, serve as transport channels in cells, act as hormones for cell signaling, support and movement of the cells and body, enzyme activity, and create antibodies in the immune system. In both types of cells the process begins when mRNA must connect to a ribosome to initiate translation, the amino acids utilized to create specific proteins are indicated by correlating codons. Amino acids are then brought to the protein chain by the tRNA until a stop codon is reached, when the protein is released. The science practice 1.2 allows the student to visualize and apply the process in which genetic code is transformed to proteins as it occurs in nature or when altered by technology such as genetic engineering, allowing the student to make predictions and address scientific questions by modeling to understand the natural process and applying it to complex scientific genetic engineering. D.Multiple Choice : Translate the following DNA sequence to a polypeptide using Figure 1. 5’ end-GTA-CAG-GAT-ATT-TGG-3’ end a. His-Leu-Val-Tyr-Stopb. His-Val-Leu-Tyr-Stop c. Val-Gln-Asp-Ile-Stopd. Asp-Val-His-Tyr-Thr E.FRQ: The structure of a protein directly relates to its function. a.a. Briefly explain how genetic information creates amino acids b.Explain the four levels of protein structure c.Explain what would happen to the structure of a protein if a mutation occurred that changed a polar amino acid to a nonpolar amino acid, and how does this affect function. Figure 1

41 Multiple Choice: C Explanation: Once the DNA strand has been transcribed in the 5’ to 3’ direction by the RNA Polymerase and it is a mature mRNA strand the code will be GUA CAG GAU AUU UGG. This is then matched to the codon chart to find the corresponding amino acids. FRQ: (a)DNA is the building block for life; it contains information that allows it to replicate itself and synthesize proteins that are necessary to live. DNA is made up of four basic nucleotides: Adenine, Guanine, Thymine, and Cytosine. In order to transfer the genetic information stored in DNA to amino acids to make polypeptides, DNA must be transcribed to RNA by the enzyme RNA Polymerase. RNA Polymerase takes one strand of DNA and transcribes it into RNA by creating an RNA strand that has the nucleotide bases A, U, C, and G from the DNA nucleotide bases of A, T, C, and G. These nucleotide bases, in groups of three, create codons that are the blueprint for an amino acid. The type of amino acids that are made depend on the codon and if there are any mutations. If a wrong based is transcribed, a wrong codon will be transcribed, and a wrong amino acid will be translated and could result in a totally different protein. (b)The codons that make an amino acid complete the primary structure of a protein, a string of amino acids held together by a covalent bond. Since structure constantly relates to function, the level of structure that a protein has determines its function within the body. The secondary structure is how the amino acid strand coils up based on the hydrogen bonds, creating an alpha helix or beta sheet structure. The tertiary structure has one polypeptide chain backbone and one or more protein secondary structures attached. Finally, the quaternary, not as common, is simply the number of multiple folded subunits of proteins. (c)When something is polar it means that the atoms interact with water, meaning they are hydrophilic. When something is nonpolar the atoms are hydrophobic and retract from water. If a polar amino acid was exchanged with a nonpolar amino acid, then the new amino acid would retract towards the inside of the polypeptide to get away from the aqueous environment, and thus changes the secondary structure and consequently all structures after that as well. The nonpolar amino acid would shift to the inside of the polypeptide because of its hydrophobic qualities, where as the polar amino acids can sit on the outside of the amino acid to protect the nonpolar amino acids from the aqueous environment. With the new change in shape the protein can no longer function as it originally did.

42 SP 6.2: The student can construct explanations of phenomena based on evidence produced through scientific practices. LO 3.34: The student is able to construct explanations of cell communication through cell-to-cell direct contact or through chemical signaling. Explanation: Cells communicate by cell-to-cell contact primarily via cell junctions and/or communication/recognition between surface molecules. Both animals and plants have cell junctions that allow molecules to pass readily between adjacent cells without crossing plasma membranes. Gap junctions are formed between animal cells, whereas plasmodesmata are specific to plants cells. Direct contact between membrane-bound cell surface molecules is particularly important in the immune response; for example MHC molecules help helper T cells recognize APCs. Cellular communication occurs over a short (paracrine) or long (endocrine) distance. However, the means of achieving the desired response varies with respect to proximity. In both local and long-distance signaling, only specific target cells recognize and respond to a given chemical signal. Cells communicate over short distances by using local regulators (e.g. a growth factor or a neurotransmitter) that target cells in the vicinity of the emitting cell. Chemical signals released by one cell type can travel long distances to target cells of another cell type. Both animals and plants use chemicals called hormones for long-distance signaling. Steroid hormones may reach virtually all body cells because they’re nonpolar and can easily pass through the membrane. When a cell encounters a signal, the signal must be recognized by a specific receptor molecule, and the information it carries must be transduced inside the cell to illicit an appropriate response. MC Question: Why is it important for immune-system cells to communicate with each other through cell- to-cell contact? A.Immune cells must form gap junctions to exchange specific antibodies for the bacterial antigen that initiated the response. B.Cytokines and hormones must bind to surface receptors on certain cells to activate killer T cells or stimulate B cells/antibody production. C.MHC proteins carry bits of cellular material to the cell surface so that a subgroup of T cells know when a cell is infected. D.The response is more localized because of the limited amount of antibodies produced via clonal selection. FRQ: During an infection, different immune cells must mount a coordinated defense against foreign invaders. This requires communication. a.How can two immune-system cells communicate with each other directly? b.Given that cytokines help activate B cells and cytotoxic T cells, what would happen in the presence of a cytokine inhibitory factor? Such as.. c.Describe and discuss the role of MHC molecules with respect to cellular communication. http://biology- forums.com/gallery/medium_88714_10_11_13_5_47_42. jpeg

43 Answer Key MC Question: Why is it important for immune-system cells to communicate with each other through cell-to-cell contact? A.Immune cells must form gap junctions to exchange specific antibodies for the bacterial antigen that initiated the response. B.Cytokines and hormones must bind to surface receptors on certain cells to activate killer T cells or stimulate B cells/antibody production. C.MHC proteins carry bits of cellular material to the cell surface so that a subgroup of T cells know when a cell is infected. D.The response is more localized because of the limited amount of antibodies produced via clonal selection. Explanation: Gap junctions are specific to animal cells and serve to transport ions and small molecules. The release of cytokines and hormones to alert other cells/activate killer T cells or stimulate B cells is an example of chemical signaling instead of direct. The response is more localized during synaptic signaling because there are actually very few neurotransmitters. C is correct because MHC proteins are used for cell identity. They constantly carry bits of cellular material from the cytosol to the cell surface. This provides a “snapshot” of what is going on inside the cell. The MHCs give the surface of cells a unique label and ultimately B and T cells can identify and recognize them as a friend or foe. FRQ: During an infection, different immune cells must mount a coordinated defense against foreign invaders. This requires communication. a.How can two immune-system cells communicate with each other directly? b.Given that cytokines help activate B cells and cytotoxic T cells, what would happen in the presence of a cytokine inhibitory factor? Such as IL-10. c.Describe and discuss the role of MHC molecules with respect to cellular communication. a.Immune cells can communicate with each other directly by binding to receptors on each other’s surfaces. B cells are known for making antibodies, which then bind to an antigen and mark the antigen for destruction by other immune system cells. T cells recognize small fragments of antigens that are bound to normal cell-surface molecules (MHC). No credit for secreted factors (i.e. cytokines, chemokines..) b.If cytokines weren’t released, communication among the immune system cells would be severely hindered. Cytokines are responsible for alerting others that an enemy is approaching, similar to a soldier sounding his bugle in an effort to efficiently and quickly inform the frontline. As a result, T cells with the correct complimentary receptor wouldn’t be cloned so that a different cytokine may ultimately be released, activating killer T cells or stimulating B cells/antibody production. c.MHC proteins are used for cell identity. They constantly carry bits of cellular material from the cytosol to the cell surface. This provides a “snapshot” of what is going on inside the cell. The MHCs give the surface of cells a unique label and ultimately B and T cells can identify and recognize them as a friend or foe.

44 Learning Objective 3.35: The student is able to create representation(s) that depict how cell-to-cell communication occurs by direct contact or through chemical signaling. Scientific Practice 1.1: The student can create representations and models of natural or man-made phenomenon and systems in the domain. Explanation: Cell-to-cell communication occurs by direct contact through either gap junctions in animal cells or plasmodesmata in plant cells. They allow movement of molecules between adjacent cells without crossing plasma membranes. In gap junctions, chemical and electrical signals can be passed between animal cells. The molecules must be small, such as ions or molecules that are smaller. Plasmodesmata are tunnels of cytoplasm between two plant cells’ plasma membrane and they allow material to be transported from cell to cell. They allow the movement of ions, amino acids, sugars, small proteins (such as transcription factors) and microRNA. Immune cells such as antigen-presenting cells, helper T-cell and killer T-cells interact by direct contact as well. Cell-to-cell communication occurs through chemical signaling in three different ways in organisms: synaptic, paracrine and hormonal. Cells can communicate over short distances by using local regulators that target cells that are close to the cell that is releasing signals. Synaptic signaling travels over short distances by using local regulators and occurs between junctions of nerve cells or between nerve and muscle cells. They use neurotransmitters that cross the space between these cells to stimulate or inhibit a nerve impulse or muscle contraction. Paracrine signaling travels short distances by using a local regulator as well and has cells that secrete substances that affect only nearby cells, which is helpful because the substances are either readily absorbed by adjacent cells or quickly broken down in the extracellular fluid. The various growth factors that are crucial to development are an example of paracrine signals. The plant immune response, quorum sensing in bacteria and morphogens in embryonic development all communicate over short distances by using local regulators. If a signal needs to travel a long distance, signals are released by one cell types and travels to target cells of another cell type. Hormonal signaling is a type of long distance signaling where endocrine cells secrete endocrine signals (hormones) that release signaling molecules into body fluids. These are specific and can travel long distances through the body, which provides a mechanism for distributing signals throughout an multicellular organism. Insulin, human growth hormones, thyroid hormones, testosterone and estrogen are all examples of many hormones that communicate through hormonal signaling. Free Response Question: Cell-to-cell communication in multicellular organisms allows cells to receive signals from other cells. Illustrate and explain the process leading to reception, the cell-to-cell communication, for each of the following molecule or hormone based on figure 11.4 inside the human body: a)Morphogen, a signaling molecule that that forms a concentration gradient in embryonic development b)Serotonin, a neurotransmitter that contributes to emotion and mood, particualrly depression c)Estrogen, the primary female sex hormone that is responsible for development and regulation of the female reproductive system and secondary sex characteristics Campbell Reece Biology Seventh Edition Figure 11.4 (page 202 )

45 Multiple Choice Question: Auxin is a growth-promoting chemical that is a primary plant hormone. It is produced in either the embryo of seed or apical meristems. Auxin is growth-promoting because the wherever the concentration of auxin is, the cells in that area elongate. Considering that the flame is the only light source, which figure accurately illustrates how cell-to-cell communication of the hormone auxin contributes to apical dominance and phototropism when the chemical is distributed unevenly within a plant shoot? (the auxin is colored beige) a) b) c)d)

46 Explanation a)This is an example of paracrine signaling. In a secreting cell, a secretory vesicle discharges the morphogen signal (the local regulator) into the extracellular fluid. It then travels to the target cell where they form a concentration gradient along the way that prefigures the pattern of embryonic development. b)This is and example of synaptic signaling. An electrical signal along a nerve cell triggers the release of serotonin. The neurotransmitter diffuses across the synapse to attach to receptors on the target cell to stimulate it and release the mood-altering response. An over-abundance of serotonin is contributed to depression. c)This is an example of hormonal signaling. Inside an endocrine cell, estrogen is secreted into blood vessels and the hormone travels throughout the bloodstream to target cells, with the ability to reach mostly all body cells. When the estrogen reaches it target cells, the response regulates the female reproductive system and secondary characteristics. Explanation This question is based off of an experiment conducted by Frits Went in 1926 that identified how a growth-promoting chemical causes a coleoptile to grow towards light. Went placed coleoptiles in the dark and removed their tips. He put some tips on agar blocks that he predicted would absorb the chemical. For the control of the experiment, Went placed a block that lacked the chemical. On the other coleoptiles, he placed blocks containing the chemical and distributed the chemical evenly or offset to increase the concentration on one side. The coleoptile grew straight if the chemical was distributed evenly and curved away from the side of the block (toward the light, although grown in the dark) when the chemical was distributed unevenly. Went concluded that a coleoptile curved towards light because its dark side a high concentration of the growth-promoting chemical which he named auxin. The role of auxin illustrates cell-to-cell communication because of the signals the chemical emits. Auxin causes cells to elongate to wherever a light source is located, which wouldn’t occur if it weren’t for the chemical signaling of auxin. This chemical signaling shows plant cells communicating within a plant to grow toward a light source where photosynthesis could occur. Free Response Question: Cell-to-cell communication in multicellular organisms allows cells to receive signals from other cells. Illustrate and explain the process leading to reception, the cell- to-cell communication, for each of the following molecule or hormone based on figure 11.4 inside the human body: a)Morphogen, a signaling molecule that that forms a concentration gradient in embryonic development b)Serotonin, a neurotransmitter that contributes to emotion and mood, particularly depression c)Estrogen, the primary female sex hormone that is responsible for development and regulation of the female reproductive system and secondary sex characteristics a) Morphogen Accurate Illustrations b) Serotonin c) Estrogen Multiple Choice Question: Auxin is a growth-promoting chemical that is a primary plant hormone. It is produced in either the embryo of seed or apical meristems. Auxin is growth- promoting because the wherever the concentration of auxin is, the cells in that area elongate. Considering that the flame is the only light source, which figure accurately illustrates how cell- to-cell communication of the hormone auxin contributes to apical dominance and phototropism when the chemical is distributed unevenly within a plant shoot? (the auxin is colored beige) a)This is incorrect. Due to the auxin being located in the tip of the shoot, it would be impossible for auxin, a growth-promoting chemical, to grow straight and not towards the light source. If the auxin was evenly distributed, it would grow straight. b)This is incorrect. While one stem of the shoot is growing towards the flame, the other is not, which, if auxin is located in the tip, would be impossible. Both stems should be growing towards the light source. c)This is correct. Auxin, a growth-promoting chemical, when distributed unevenly (in the tip) will grow towards a light source because of the higher concentration of auxin in the tip of the shoot. d)This is incorrect. The uneven distribution of auxin will grow towards light because of the high concentration of auxin, which is a growth-promoting chemical that grows towards the area where photosynthesis could occur. This is an example of negative phototropism, but auxin’s role controls positive phototropism.

47 LO 3.38: The student is able to describe a model that expresses key elements to show how change in signal transduction can alter cellular response. SP 1.5: The student can reexpress key elements of natural phenomena across multiple representations in the domain. Explanation: A signal transduction pathway begins when a signal molecule, also known as a ligand, binds to a receptor, there can be G-protein linked receptors which means once it binds to the receptor, the receptor will change shape and attracts an inactive G -Protein then GTP displaces GDP and activates the G-Protein. Also another receptor is tyrosine kinase receptors and when it is bound to, the receptor forms a dimer and the cytoplasmic tails phosphorylate each other and ATP is hydrolyzed to activate the receptor which can than activate other relay proteins, those are only just a few of many other receptors with others such as intracellular receptors and ligand gated ion channels. However after reception leads to transduction which is when the signal is transformed in the cell to exhibit a certain response. Most responses are activated by a relay molecule activating the phosphorylation cascade which is when one kinase would activate another. Also during this process phosphate groups of proteins are being removed by protein phosphatases so the proteins will be inactive and be able to be reused. Also can be done through second messengers where adenylyl converts ATP to cAMP and acts as an allosteric activator. Additionally other second messengers such as Ca +2, IP 3 (inositol triphosphate), and DAG (diacyl glycerol) all can be ways of transduction. Than lastly there is response which can be shown in many ways such as cytoplasmic or nuclear signal amplification. However if anything within the process is altered it can ultimately change the cellular response of the cell. Such as the people with Type 2 Diabetes, when there is an inhibitor to the signal molecule which ultimately changes the response by not having the release of insulin. However if this was human without type 2 diabetes than there would be no inhibitor to the signal molecule and insulin could be released and glucose levels could be regulated normally. MC Question: In mitosis several signal transduction pathways use different kinases. A foreign chemical is introduced to a cell that allosterically inhibits these kinases and alters the activation site. With the binding of the kinases how is mitosis affected? A) Mitosis would not be affected. B) A different cellular response will occur because the kinases are being inhibited. C) Mitosis would completely stop and no cellular response will occur. D) Mitosis would continue at a slower rate, because kinases are just an enzyme used to speed up the reactions. FRQ: This figure displays a signal transduction pathway. a.List and describe each step and what occurs at each of the labeled numbers. b.Identify and explain the function painkillers and how it is able to change the cellular response? c.How is cancer an example of a change in signal transduction pathway altering cellular response? 213

48 Answer Key MC Question: In mitosis several signal transduction pathways use different kinases. A foreign chemical is introduced to a cell that allosterically inhibits these kinases and alters the activation site. With the binding of the kinases how is mitosis affected? A) Mitosis would not be affected. B) A different cellular response will occur because the kinases are being inhibited. C) Mitosis would completely stop and no cellular response will occur. D) Mitosis would continue at a slower rate, because kinases are just an enzyme used to speed up the reactions. Explanation: Mitosis would be completely stopped because active protein kinases would be needed to activate a cellular response. During the transduction stage, a phosphorylation cascade must occur. During the phosphorylation kinases are continuously being turned on and off by protein phosphotases. Therefore if the kinases are being inhibited there will be no transduction of the ligand which means there will be a change of response, which is no response is exhibited. FRQ: This figure displays a signal transduction pathway. a.List and describe each step and what occurs at each of the labeled numbers. b.Explain the function of anti-histamines and how it is able to change the cellular response? c.How is cancer an example of a change in signal transduction pathway altering cellular response? a.1: A signal molecule also known as a ligand binds to a receptor, the receptor will either alter it’s shape or form a dimer and induce a response within the cell. 2: This stage is transduction pathway and where the signal response has the ability to be amplified either through protein phosphorylations, cylic AMP, or Ca +2 phosphorylations activate protien kinases which turn inactive proteins to become active through the phosphate of ATP to display a cellular response. cAMP can be an second messangers when ATP is added they can produce a response. Lastly Ca +2 ions are able to also act as second messangers when IP 3 binds to the ligand gated ion channel calcium ions can be released to create a cellular response. 3: This final stage is the response where a response can be cytoplasmic amplified such as glycogen can be emitted in mass amounts into the body. Also there is nuclear amplification is when it induced RNA polymerase to either transcribe to inihibit transcription of a gene. b.Anti-histamines are able to alter the cellular response when toxin, such as an antigen binds to the IgE on a cell it causes a release of histamines. However when the antihistamines binds to the receptors which releases the histamines it is able to change the response by not exhibiting histamines. c.Growth factor signals cultivate on the cell and signaling pathways such as tyrosine kinase that involves IP 3 is altered which causes an inhibition of apoptosis and cell death which causes a change in response of uncontrollable cell growth and division of cells thus causing cancer.

49 LO 1.29: The student is able to describe the reason for revision of scientific hypotheses of the origin of life on Earth. SP 6.3: The student can articulate the reason that scientific explanations and theories are refined or replaced. Explanation: Early Earth’s atmosphere consisted of carbon-dioxide, carbon-monoxide, water, methane, nitrogen, and little oxygen. The presence of available free energy made it possible for inorganic molecules to be synthesized to organic molecules. These molecules served as building blocks, or monomers, for more complex molecules like amino acids and nucleotides. In the 1920’ s Oparin and Haldane hypothesized that early Earth favored reactions that formed organic compounds from inorganic compounds. They suggested that the early oceans were a solution of organic molecules, a “primitive soup” from which life arose. Then in 1953 the Miller and Urey experiment tested and proved that all twenty amino acids needed for life would be able to be synthesized in early Earth’s atmosphere, proving Oparin and Haldane correct. Sydney Fox connected monomers to make polymers by dripping monomers on hot sand, clay, or rocks which formed polypeptide bonds between them by abiotic means. Protobionts abiotically produced molecules surrounded by a membrane. Stable protobionts, coacervate, self-assemble when a suspension of macromolecules is shaken. The joining of monomers produced polymers that were able to store, replicate, and transfer information by using what is believed to be the first genetic material, ribonucleic acid (RNA). Primitive cells imprecisely reproduced and had a simple metabolism. Multiple Choice Question: Looking at the figure, what did the Miller and Urey experiment prove to be true about early earth? A.Volcanic activity caused the formation of organic molecules. B.The conditions of early earth could spontaneously form all twenty amino acids needed for life. C.Bacteria were the first living organisms on earth. D.Ammonia and methane were not needed to from life on early earth. E.DNA was the first genetic material. Free Response Question: Guadalupe thinks that Miller and Urey’s experiment wasn’t possible in early Earth’s conditions and disproved Oparin and Haldane’s hypothesis. Emilie believes that Miller and Urey’s experiment was possible in early Earth’s conditions and proved Oparin and Haldane’s hypothesis correct. Determine whether Guadalupe or Emilie is correct and explain why they are correct. Source: http://www.slideshare.net/MissReith/chap-27-evolution

50 Answer Key Multiple Choice Question: What did the Miller and Urey experiment prove to be true about early earth? A.Volcanic activity caused the formation of organic molecules. B.The conditions of early earth could spontaneously form all twenty amino acids needed for life. C.Bacteria were the first living organisms on earth. D.Ammonia and methane were not needed to from life on early earth. E.DNA was the first genetic material. Explanation: Miller and Urey’s experiment proved that the conditions of early Earth could spontaneously form all twenty amino acids needed for life by creating a model of early Earth’s atmosphere. Free Response Question: Guadalupe thinks that Miller and Urey’s experiment wasn’t possible in early Earth’s conditions and disproved Oparin and Haldane’s hypothesis. Emilie believes that Miller and Urey’s experiment was possible in early Earth’s conditions and proved Oparin and Haldane’s hypothesis correct. Determine whether Guadalupe or Emilie is correct and explain why they are correct. Emilie’s statement is correct, Miller and Urey’s experiment was possible in early Earth’s conditions and proved Oparin and Haldane’s hypothesis correct. Early Earth’s atmosphere consisted of carbon-dioxide, carbon-monoxide, water, methane, nitrogen, and little oxygen. Lightning, UV radiation, volcanic activity, and meteorites bombarded the planet. Miller and Urey set up a closed system to stimulate conditions they thought to resemble early Earth. The primeval sea was simulated by a warmed flask of water. The atmosphere created in the experiment consisted of hydrogen, methane, ammonia, and water vapor. Lightning was simulated by sparks. A condenser cooled the atmosphere, raining water and other compounds into a miniature sea. Oparin and Haldane hypothesized that early Earth’s atmosphere could have formed simple molecules from organic compounds. They suggested that the early oceans were a solution of organic molecules, a “primitive soup” from which life arose. Miller and Urey’s experiment tested this by setting up a system that mimicked early Earth’s atmosphere and conditions which yielded a variety of organic compounds and all amino acids found in organisms today, proving Oparin and Haldane correct.

51 LO2.7: Students will be able to explain how cell size and shape affect the overall rate of nutrient intake and rate of waste elimination SP6.2: Organisms must exchange matter with the environment to grow, reproduce During cells growth, its relative surface area ratio decreases and it’s demand for material resources increases because more cellular structures are necessary to adequately exchange materials and energy with the environment. The surface area must be large enough to efficiently exchange materials. The smaller the cell, the more favorable of a surface area to volume ratio for exchange of materials with the environment (done through diffusion). The volume determines how much enters and leaves the cell while surface area determines the rate at which nutrients are taken in or excremented. Volume increases faster than surface area due to the rate that nutrients are able to be taken in and waste taken out is not fast enough to accommodate the size of the cell. This means that the volume will increase much faster than the surface area. This puts an upper limit on the size of a cell, because if the cell volume gets too big, there won't be enough membrane to transport the amount of food in and wastes out to support that large cell size.Cells produce atp(needed for survival) via cellular respiration. The process of cellular respiration incorporates the exchange of sodium and potassium in an outside the cellular membrane. Sodium and potassium are gained within the cell through consumption of matter and organic molecules. Inhibition of nutrient intake can inhibit the cell elimination. https://www.google.com/url?sa=i&rct=j&q=&esrc=s&source=images& cd=&cad=rja&uact=8&ved=0ahUKEwio05KY- 6DMAhXH7CYKHT- nDYAQjB0IBg&url=https%3A%2F%2Fwww.quora.com%2FW hat-is-an-explanation-for-the-following-statement-the-plasma- membrane-of-each-of-the-epithelial-cells-lining-the-villi-forms- cytoplasmic-extension-called-microvilli-the-side-facing-the- lumen&bvm=bv.119967911,d.eWE&psig=AFQjCNFCur1E1rt3 nZ1M-3kRD2OTLn3DGQ&ust=146136903803492 MC Question:Leo is conducting a lab experiment where he is examining the intake rate of sodium and potassium within three different eukaryotic cells. Cell A’s surface area is larger than its volume. Cell B and C’s volume is larger than its surface area. However, cell C lacks membrane-enclosed organelles. Which of the following cells will most efficiently pursue nutrient intake and waste elimination? A.Cell A B.Cell B C.Cell A and B D.Cell C FRQ:Eukaryotic cells have selectively permeable membranes that allows nutrients(i.e glucose) to be taken in and waste excremented through osmosis and diffusion while inhibiting large/unwanted molecules(i.e toxins) from entering. a.)Describe the effect how cell size and shape affect the overall rate of nutrient intake and rate of waste elimination. b.) What advantage does eukaryotic cells have over prokaryotic cells?

52 ●MC Question: Leo is conducting a lab experiment where he is examining the intake rate of sodium and potassium within three different eukaryotic cells. Cell A’s surface area is larger than its volume. Cell B and C’s volume is larger than its surface area. However, cell C lacks membrane-enclosed organelles. Which of the following cells will most efficiently pursue nutrient intake and waste elimination? A.Cell A Explanation:The correct answer is C because cells with larger volumes to surface area ratios are optimal in nutrient intake and waste elimination.The volume determines how much enters and leaves the cell while surface area determines the rate at while nutrients are taken in or excremented. B.Cell B C.Cell B and C D.Cell C ●Eukaryotic cells have selectively permeable membranes that allows nutrients(i.e glucose) to be taken in and waste excremented through osmosis and diffusion while inhibiting large/unwanted molecules(i.e toxins) from entering. Describe the effect how cell size and shape affect the overall rate of nutrient intake and rate of waste elimination. What advantage does eukaryotic cells have over prokaryotic cells? a.)The surface area must be large enough to efficiently exchange materials. The smaller the cell, the more favorable of a surface area to volume ratio for exchange of materials with the environment (done through diffusion). The volume determines how much enters and leaves the cell while surface area determines the rate at which nutrients are taken in or excremented. b.) Prokaryotes are usually much smaller and don't have a nucleus or any other membrane-bound organelles. Thus meaning they can’t pursue all the functions of eukaryotes(i.e forming tissue). Eukaryotes also have nuclear membranes containing genetic material and more protection.


Download ppt "AP Review Projects 1 st Period. Explanation: Negative Feedback is a primary mechanism of homeostasis, the condition of the body in which internal elements,"

Similar presentations


Ads by Google